Answer (B) is correct . The equipment should be recorded at $360,000. This amount equals $390,000 (FC1,000,000 x $0.39 spot rate at 2/15/Yr 2) minus the $30,000 ($19,600
Answer (A) is incorrect because The amount that would have been recognized if the equipment had been delivered on 11/15/Yr 1 is $350,000. Answer (C) is incorrect because The amount that would have been recognized if the firm commitment had not been hedged is $390,000. Answer (D) is incorrect because The amount of $420,000 equals $390,000 plus the $30,000 balance in the firm commitment liability account.
|